You are on page 1of 9

Solutions to Problems in Peskin and Schroeder,

An Introduction To Quantum Field Theory


Homer Reid
November 16, 2002
Chapter 2
Problem 2.1
(a) The Lagrangian density is
L =
1
4
F

.
Since F
ij
= F
ji
there are really only 6 terms in the sum:
=
1
2
_
F
01
F
01
+ F
02
F
02
+ F
03
F
03
+ F
12
F
12
+ F
13
F
13
+ F
23
F
23
_
.
Also, F
ij
= F
ij
if one of i or j is zero, and F
ij
= +F
ij
if both i, j are nonzero,
so we may write
= +
1
2
_
F
2
01
+ F
2
02
+ F
2
03
F
2
12
F
2
13
F
2
23

=
1
2
_
(d
0
A
1
d
1
A
0
)
2
+ (d
0
A
2
d
1
A
2
)
2
+ (d
0
A
3
d
1
A
3
)
2
(d
1
A
2
d
2
A
1
)
2
(d
1
A
3
d
3
A
1
)
2
(d
1
A
2
d
2
A
1
).
2

.
Evidently we have
L
A

= 0
L
(

)
=
_
(

), = 0
(

), = 0.
= F

= F

.
1
Homer Reids Solutions to Peskin and Schroeder Problems: Chapter 2 2
Hence the Euler-Lagrange equation is
0 =

L
(

)
=

.
Identifying E
i
= F
0i
and B
k
=
ijk
F
ij
, the Euler-Lagrange equations be-
come that

1
E
1
+
2
E
2
+
3
E
3
= 0 ( E = 0)

ijk

i
B
j
=
0
E
k
_
B =
E
t
_
.
which are two of Maxwells equations.
(b) First lets review this whole Noether theorem business. Suppose we make
a change, parameterized by a small parameter , under which the elds in the
Lagrangian undergo the transformations

i
(x)

i
(x) =
i
(x) +
i
(x) (1)
where the
i
(x) are elds themselves. This transformation does not aect the
equations of motion if the Lagrangian of the system is only modied by the
addition of the divergence of a four-vector, i.e. if
L L

= L +

(x). (2)
On the other hand, computing directly from (1) we can also write the trans-
formed Lagrange density as
L L

= L +

i
_
L

i
(
i
) +
L
(

i
)
(

i
)
_
= L +

i
_
L

i
(
i
) +

_
L
(

i
)

i
_

L
(

i
)
_
(where we used the chain rule, (fg) = fg + gf)
= L +

i
_

_
L
(

i
)

i
_
+
i
_
L

L
(

i
)
__
= L +

i
_
L
(

i
)

i
_
where we used the Euler-Lagrange equation in going to the last line. Comparing
with (2) we see that

i
L
(

i
)

i
=

Homer Reids Solutions to Peskin and Schroeder Problems: Chapter 2 3


or

= 0, j

= J

i
L
(

i
)
.
In the specic case of space-time translations, the symmetry in question is x

+ a

. We consider only translations in one dimension, so that only the th


component of a

is nonzero, and we put a

= a x

. The Lagrangian transforms


according to
L L

= L + a

L
To rewrite this in the form of (2), we dene the vector J by setting its th
component equal to L and the rest of its components to zero:
= L + a

, J

= L

.
On the other hand, the components of the elds themselves transform according
to
A

(x) A

(x) = A

(x) + a

(x)
so in this case

, = 0, 1, 2, 3. Then the conserved current corre-


sponding to translations in the x

direction is
j

= J

L
(

= L

= L

(3)
This is evidently not symmetric in the and indices. Instead we consider

= T

(F

)
= T

) F

)
The vanishing of the middle term here is the Euler-Lagrange equation we worked
out in part one, so we just get
= T

)
Substituting back in from equation (3) we nd
= L

) F

)
= L

)
= L

= L

+ F

which is symmetric in the and indices.


Homer Reids Solutions to Peskin and Schroeder Problems: Chapter 2 4
Problem 2.2 The complex scalar eld.
Part a. The momentum conjugate to is
=
L
(
t
)
=

(
t
)
_
(
t

)(
t
) (

) () + m
2

_
=
t

.
Similarly, the momentum conjugate to

is

=
t
. Then the canonical
commutation relations are
[,
t

] = [

,
t
] = i.
The Hamiltonian density is
H = (
t
) +

(
t

) L
= (
t
)(
t

) + (
t

)(
t
) (
t
)(
t

) + () (

) + m
2

= (
t
)(
t

) + () (

) + m
2

(4)
Expressed in terms of the variables appropriate for the Hamiltonian description,
we have
=

+ () (

) + m
2

. (5)
The equation of motion for is
i

t
(x

) = [H, (x

)]
=
__
H(x

) d
3
x

, (x

)
_
=
_
d
3
x

[H(x

), (x

)]
The only term in (5) that fails to commute with is the term containing :
=
_
d
3
x[(x

), (x

)]

(x

)
= i
_
d
3
x(x

x)

(x

)
= i

(x

). (6)
Before computing the equation of motion for

we rewrite the Hamiltonian


a little:
H =
_
Hd
3
x
=
_
_

+ (

) () + m
2

_
d
3
x
Homer Reids Solutions to Peskin and Schroeder Problems: Chapter 2 5
Integrating by parts on the middle term and expecting that the surface term
vanishes since the elds vanish at innity (but how does this work since the
elds are operators and not numbers?) we obtain
=
_
_

(
2
+ m
2
)
_
d
3
x.
Commuting this with

, we obtain
i

t

(x

) = i
_
(x x

)
_
(
2
+ m
2
)(x

d
3
x

= i(
2
+ m
2
)(x

).
Combining this with (6) we obtain
_

2
m
2

= 0,
i.e. the Klein-Gordon equation.
Part b. Since is no longer purely real, the coecient of e
ipx
in its Fourier
expansion no longer need equal the adjoint of the coecient of e
ipx
. We put
(x, t) =
_
d
3
p
(2)
3
_
2
p
(a
p
e
ipx

+ b

p
e
ipx

(x, t) =
_
d
3
p
(2)
3
_
2
p
(a

p
e
ipx

+ b
p
e
ipx

)
Then we have
=
t

(x, t) = i
_
d
3
p
(2)
3
_

p
2
(a

p
e
ipx

b
p
e
ipx

)
and
[(x

), (y

)] = i
_
d
3
p d
3
p

2 (2)
6
_

p
_
[a
p
, a

p
]e
i(px

x)
[b

p
, b
p
]e
i(px

)
_
= i(x y)
where we have assumed
[a
p
, a

p
] = [b
p
, b

p
] = (p p

)
and all other commutators vanish. So there are two dierent particles of mass
m, one created by a

and one created by b

.
Homer Reids Solutions to Peskin and Schroeder Problems: Chapter 2 6
Part c. The symmetry in this case is with respect to the global phase of .
We can take e
i
(1 + i) without aecting the dynamics.
The corresponding conserved Noether current is given by
j

= J

i
L
(

i
)

i
.
Under this symmetry operation the Lagrangian doesnt change at all, so J

= 0.
The generators of changes in the elds are = i and

= i

, while
L/(

) =

and vice versa. Then we obtain


j

= i(

).
Integrating the 0 component of this, we obtain the conserved charge:
Q = i
_
d
3
x(

).
The ipped sign and extra factor of 2 as compared with the expression in P+S
are obviously just matters of convention.
Expressed in terms of creation and destruction operators, we have
Q = i
_ _ _
d
3
xd
3
p d
3
p

2(2)
6
_
a
p
a

p
e
ix

(p

)
a
p
b
p
e
ix

(p

+p

)
b

p
a

p
e
ix

(p

+p

)
b

p
b
p
e
ix

(p

)
_
c.c.
The integral over x cancels a factor of (2)
3
and gives us some functions:
=
i
2
_ _
d
3
p d
3
p

(2)
3
_
a
p
a

p
(p

p) a
p
b
p
(p

+p)
b

p
a

p
(p

+p) b

p
b
p
(p

p)
_
c.c.
=
i
2
_
d
3
p
(2)
3
_
a
p
a

p
a
p
b
p
+ b

p
a

p
b

p
b
p
_
c.c.
=
_
d
3
p
(2)
3
_
a

p
a
p
b

p
b
p
_
.
So the a and b particles have the opposite sign of the charge.
Part d. Working in full generality from the start, we imagine we have n
complex scalar elds, whose real and imaginary parts we assemble into a column
vector of 2n real elds:
=
_
_
_
_
_
_
_

1r
(x)

1i
(x)

2r
(x)
.
.
.

ni
(x)
_
_
_
_
_
_
_
.
Homer Reids Solutions to Peskin and Schroeder Problems: Chapter 2 7
The Lagrangian is
L =

n
_

n
m
2

n
_
= (

)
t
(

) m
2

Since the elds all have the same mass, we can arbitrarily redene which
elds we call which without aecting the Lagrangian dynamics. In fact, the
Lagrangian remains invariant under any morphing of the elds into linear com-
binations of themselves, so long as the length of the vector remains constant.
All transformations that satisfy this condition can be represented by multiplying
by a matrix M SO(2n), i.e.

= M.
Now, any matrix M SO(2n) may be written
M = e
i(1J
1
+2J
2
++mJ
m
)
where the
i
are arbitrary real parameters and the J
i
are matrices containing
exactly two nonzero entries, a +1 in one of the upper diagonal entries and a -1
in the transposed entry. Since there are 2n(2n 1)/2 upper diagonal slots in a
2n 2n matrix, there are m = 2n(2n 1)/2 generator matrices J
i
for SO(2n).
For example, for n = 2 we have
J
1
=
_
_
_
_
0 1 0 0
1 0 0 0
0 0 0 0
0 0 0 0
_
_
_
_
J
2
=
_
_
_
_
0 0 1 0
0 0 0 0
1 0 0 0
0 0 0 0
_
_
_
_
J
3
=
_
_
_
_
0 0 0 1
0 0 0 0
0 0 0 0
1 0 0 0
_
_
_
_
J
4
=
_
_
_
_
0 0 0 0
0 0 1 0
0 1 0 0
0 0 0 0
_
_
_
_
J
5
=
_
_
_
_
0 0 0 0
0 0 0 1
0 0 0 0
0 1 0 0
_
_
_
_
J
6
=
_
_
_
_
0 0 0 0
0 0 0 0
0 0 0 1
0 0 1 0
_
_
_
_
.
Hence the internal symmetry group of the Lagrangian for n complex scalar elds
is a 2n(2n 1)/2-parameter group, and there are thus 2n(2n 1)/2 conserved
currents. For example, for 2 complex scalar elds there are 4 3/2 = 6 conserved
currents.
To nd the conserved current associated with the kth parameter, we consider
the transformation
e
iJ
k
(1 + iJ
k
).
The Lagrangian is unchanged under this transformation, so again J

= 0. The
generator of transformations in the mth eld (where m = 1, 2, , 2n counts
separately over the real and imaginary components of the n complex elds) is

m
= i

p
J
k
mp

p
= i
_
J
k

_
m
Homer Reids Solutions to Peskin and Schroeder Problems: Chapter 2 8
(Note that the k superscript on J selects which of the matrices were talking
about, while the m, p subscripts select which entry of that matrix we want). On
the other hand, as before,
L/(

m
) =

m
Hence the conserved Noether current corresponding to the kth parameter in
the parameterization of SO(2n) is
j

k
=

m
L
(

m
)

m
= i

m
(

m
)
_

p
J
k
mp

p
_
We could also write this as
= i

m
(

m
)
_
J
k

_
m
.
The conserved charge density corresponding to the kth parameter is the 0 com-
ponent of this four-vector:

k
= i

m
_
J
k

_
m
= i
t
J
k
.
where the ith entry of the column vector is the canonical momentum conju-
gate to the ith eld in the column vector .
Problem 2.3
Since the integral is Lorentz invariant we can choose any convenient reference
frame in which to work, and we choose the frame in which q

(x

) has
the form
q

= (0, rk).
Then the dot product between the vectors is
p

= p
0
q
0
p q = p
r
r cos .
Then in polar coordinates the integral is
D(x y) = D(q) =
1
(2)
3
_

0
p
2
dp
2
_
p
2
+ m
2
_

0
d e
iprr cos
sin
_
2
0
d
=
1
(2)
2
_

0
p
2
dp
2
_
p
2
+ m
2
_
1
1
du e
iprru
Homer Reids Solutions to Peskin and Schroeder Problems: Chapter 2 9
(where we did the integral and put u = cos in the integral)
=
1
2
2
r
_

0
p sin pr dp
_
p
2
+ m
2
. (7)
Now, Gradstein and Ryzhik equation (3.754.2) is
_

0
cos axdx
_

2
+ x
2
= K
0
(a).
Dierentiating both sides with respect to a, we obtain

_

0
a sinaxdx
_

2
+ x
2
= K

0
(a) = K
1
(a).
Comparing with (7) we nd
D(x y) =
1
2
2
m
|x y|
K
1
(m|x y|).

You might also like